PT10.S4.Q1 - What effect answer choice E has on the argument?

Ashley2018-1Ashley2018-1 Live Member
edited June 2021 in Logical Reasoning 2249 karma

Could someone explain what effect answer choice E has on the argument? Doesn’t it point out a flaw since the stimulus only talks about physicians who are too thorough and those who are not at all without considering the middle ground?

Does AC A resolve the potential issues with physicians who aren’t thorough enough?

Admin note: edited title; please use the format of "PT#.S#.Q# - [brief description]"

Comments

  • dubattdubatt Core Member
    41 karma

    Hey @Ashley25 the key here is to remember to stay on task. Specifically you want to make the conclusion that it is generally unwise for patients to have medical checkups when they don't feel well, more likely to be false. 'E' tells us that some physicians sometimes use the correct amount of thoroughness. If only some physicians out of the larger population of physicians use the correct amount of thoroughness, this does not make it less likely that the patients generally should not go get checkups. 'A', on the other hand, tells us that there are some diseases which physicians (all physicians) are able to detect early even though the patient does not not feel ill. If all physicians are able to detect early symptoms it makes the conclusion to generally not go to physicians more likely to be false. I think also remember that with strengthening and weakening questions the correct answer doesn't have to 100% destroy or strengthen the conclusion, it just needs to be enough for the conclusion to be less or more likely.

  • Ashley2018-1Ashley2018-1 Live Member
    edited June 2021 2249 karma

    @dubatt said:
    Hey @Ashley25 the key here is to remember to stay on task. Specifically you want to make the conclusion that it is generally unwise for patients to have medical checkups when they don't feel well, more likely to be false. 'E' tells us that some physicians sometimes use the correct amount of thoroughness. If only some physicians out of the larger population of physicians use the correct amount of thoroughness, this does not make it less likely that the patients generally should not go get checkups. 'A', on the other hand, tells us that there are some diseases which physicians (all physicians) are able to detect early even though the patient does not not feel ill. If all physicians are able to detect early symptoms it makes the conclusion to generally not go to physicians more likely to be false. I think also remember that with strengthening and weakening questions the correct answer doesn't have to 100% destroy or strengthen the conclusion, it just needs to be enough for the conclusion to be less or more likely.

    If A were true would the physicians who aren’t thorough be able to detect those symptoms as well? I wasn’t 100% sure since it said these physicians were “likely” to miss symptoms

  • dubattdubatt Core Member
    41 karma

    Yes 'physicians' in this case includes everyone. I think it is a safer bet to take the answer choices as they are presented and pull out the implication just from what is given. Also if you are unsure, reading all the other answer choices, they are more clearly wrong for the task. So in this case if you are not 100% convinced by 'A' but you think it is a contender eliminating all the other choices will help to pick A more confidently.

  • Ashley2018-1Ashley2018-1 Live Member
    2249 karma

    @dubatt said:
    Yes 'physicians' in this case includes everyone. I think it is a safer bet to take the answer choices as they are presented and pull out the implication just from what is given. Also if you are unsure, reading all the other answer choices, they are more clearly wrong for the task. So in this case if you are not 100% convinced by 'A' but you think it is a contender eliminating all the other choices will help to pick A more confidently.

    Thanks for responding…would you say the argument is a flawed one?

Sign In or Register to comment.